jenndg100380
Thanks Received: 0
Forum Guests
 
Posts: 18
Joined: August 03rd, 2010
 
 
 

PT36, S1, Q18 - Moderate exercise lowers the risk

by jenndg100380 Sat Aug 28, 2010 5:53 pm

Can someone please explain this to me? I just assumed that D was a throw away answer. Why wouldn't it be (A)? If you negate it and say that the study was not between exercise and blood cholesterol level, how would the conclusion be properly drawn?
User avatar
 
ManhattanPrepLSAT1
Thanks Received: 1909
Atticus Finch
Atticus Finch
 
Posts: 2851
Joined: October 07th, 2009
 
This post thanked 2 times.
 
 

Re: Q18 - Moderate exercise lowers the risk

by ManhattanPrepLSAT1 Tue Aug 31, 2010 4:32 am

Tough question.

One reason why you cannot just negate the answer choices to see which one is the assumption of the argument is that this is not a necessary assumption question. This is a sufficient assumption question and so you're looking to find the missing piece that you can add to the argument that guarantees that the conclusion will follow. Remember, the negation test only works on necessary assumption questions!

The argument can be converted to formal notation

DC ---> (ME ---> LBC)
LBC ---> LRHA
LRHA ---> LRB
-----------------
ME ---> LRB

(Notation Key: DC = data correct, ME = moderate exercise, LBC = lowers blood cholesterol, LRHA = lowers risk of hardening of the arteries, LRB = lowers risk of arterial blockage)

The argument is so close to being validly drawn as it is. If we simply assume that the data are correct, then we know that moderate exercise lowers blood cholesterol levels and at that point the argument would be validly drawn - best expressed in answer choice (D).

(A) would definitely help, but it's not really important whether that relationship or another was studied, but rather whether the data implied the relationship that moderate exercise lowers blood cholesterol. The study could have been looking at something else and still made the discovery.
(B) is irrelevant. Whether arterial blockage can be prevented says nothing as to what whether it was moderate exercise that prevented the blockage of the arteries.
(C) attempts to skip a premise, but would still not allow us to conclude that moderate exercise reduces the risk of arterial blockage, because we don't know whether the data is correct.
(E) is a negation of one of the premises and also does not allow the conclusion regarding moderate exercise to be established.

I hope that helps clear this one up!

#officialexplanation
 
peg_city
Thanks Received: 3
Forum Guests
 
Posts: 152
Joined: January 31st, 2011
Location: Winnipeg
 
 
trophy
First Responder
 

Re: PT36, S1, Q18 - Moderate exercise lowers the risk

by peg_city Sat Apr 09, 2011 6:59 pm

mshermn Wrote:Tough question.

One reason why you cannot just negate the answer choices to see which one is the assumption of the argument is that this is not a necessary assumption question. This is a sufficient assumption question and so you're looking to find the missing piece that you can add to the argument that guarantees that the conclusion will follow. Remember, the negation test only works on necessary assumption questions!

The argument can be converted to formal notation

DC ---> (ME ---> LBC)
LBC ---> LRHA
LRHA ---> LRB
-----------------
ME ---> LRB

(Notation Key: DC = data correct, ME = moderate exercise, LBC = lowers blood cholesterol, LRHA = lowers risk of hardening of the arteries, LRB = lowers risk of arterial blockage)

The argument is so close to being validly drawn as it is. If we simply assume that the data are correct, then we know that moderate exercise lowers blood cholesterol levels and at that point the argument would be validly drawn - best expressed in answer choice (D).

(A) would definitely help, but it's not really important whether that relationship or another was studied, but rather whether the data implied the relationship that moderate exercise lowers blood cholesterol. The study could have been looking at something else and still made the discovery.
(B) is irrelevant. Whether arterial blockage can be prevented says nothing as to what whether it was moderate exercise that prevented the blockage of the arteries.
(C) attempts to skip a premise, but would still not allow us to conclude that moderate exercise reduces the risk of arterial blockage, because we don't know whether the data is correct.
(E) is a negation of one of the premises and also does not allow the conclusion regarding moderate exercise to be established.

I hope that helps clear this one up!

Sorry to be a bother :)

Why is B wrong when, if it were not assumed, then we know that moderate exercise cannot prevent a blockage of the arteries?
User avatar
 
ManhattanPrepLSAT1
Thanks Received: 1909
Atticus Finch
Atticus Finch
 
Posts: 2851
Joined: October 07th, 2009
 
 
 

Re: Q18 - Moderate exercise lowers the risk

by ManhattanPrepLSAT1 Sun Apr 10, 2011 3:20 am

No problem, we love your questions! Keep 'em coming. But I think on this one, I need to turn it around and ask you how you see answer choice (B) as establishing the conclusion.

So if blockage of the arteries due to blood clots can be prevented, does that prove the conclusion that moderate exercise lowers the risk of blockage of the arteries due to blood clots. The fact that such blockage can be prevented is great! But does that mean that it was the moderate exercise that is responsible. If you see a chain of reasoning that involves answer choice (B) and establishes the conclusion, I'd love to have you show me the connection!
User avatar
 
gilad.bendheim
Thanks Received: 21
Jackie Chiles
Jackie Chiles
 
Posts: 31
Joined: August 20th, 2011
 
 
 

Re: Q18 - Moderate exercise lowers the risk

by gilad.bendheim Sun Sep 04, 2011 2:47 pm

Hi - I'm actually confused on this one as well. As far as I understand it, if the conclusion itself admits that "if the data reported in a recent study are correct," then the conclusion will follow regardless of whether the data is correct or not. Let me try to explain.

If the data IS correct:
The conclusion follows logically because it then predicts that moderate exercise will lower blood cholesterol levels.

If the data IS NOT correct:
The conclusion still follows logically, because it would then make no prediction about the relationship between exercise and cholesterol.

I'm looking at the conclusion not as a fact set, but itself as another conditional. The conditional X -> Y is logical in a number of ways. One way is if X occurs, then Y occurs (which is our definition if the data IS correct). Another way it is logical is if X does not occur, and we draw no conclusion about whether or not Y occurs (this is what we do if the data is NOT correct).

If the question asked "what must be true in order for the conclusion to be drawn," the answer would be choice B. Alternatively, if the stimulas simply said "A new study reported..." without the caveat about the data being correct, the answer would be choice B as well. But the way this was phrased, allows for the logic of the statement to remain sound whether or not the data is correct -- after all, that is the whole reason the caveat is inserted - to ensure that the logical validity of the argument is not undone simply by disproving the evidence upon which it is based.

Because of this I chose choice C, which regardless of the results of the data, would be required to make the above conclusion.

What am I missing here?

Thanks
 
timmydoeslsat
Thanks Received: 887
Atticus Finch
Atticus Finch
 
Posts: 1136
Joined: June 20th, 2011
 
This post thanked 2 times.
 
trophy
Most Thanked
trophy
First Responder
 

Re: Q18 - Moderate exercise lowers the risk

by timmydoeslsat Sun Sep 04, 2011 3:00 pm

Make sure to remember that just because something is required does not mean it is sufficient to draw a certain conclusion.

This stimulus can be seen as this:



Lower blood cholest levels ---> Lowers risk of hardening of arteries ---> Lowers risk of blockage of arteries due to blood clots

Data reported correct ---> ME lowers blood cholest levels

________________________________________________________
ME lowers risk if blockage of arteries due to blood clots.


As of now, we need something with ME to link up to that chain in the stimulus. The second conditional statement in my setup gives us such an opportunity. If the data reported is correct, we then know that ME lowers blood cholest levels, and then that sets off the first long conditional chain in my set up.

That would guarantee our conclusion of ME lowering risk of blockage of arteries due to blood clots. This idea is the last idea in the chain.

Choice C would not be sufficient to allow us to conclude that ME lowers risk of arterial blockage.
 
tzyc
Thanks Received: 0
Atticus Finch
Atticus Finch
 
Posts: 323
Joined: May 27th, 2012
 
 
trophy
Most Thankful
 

Re: Q18 - Moderate exercise lowers the risk

by tzyc Sun Mar 31, 2013 8:04 pm

What does this ";" represent??
Maybe a small issue but wonder if it has some meaning...

Thank you
 
wj097
Thanks Received: 0
Atticus Finch
Atticus Finch
 
Posts: 123
Joined: September 10th, 2012
 
 
 

Re: Q18 - Moderate exercise lowers the risk

by wj097 Fri Apr 05, 2013 2:13 am

mattsherman Wrote:(B) is irrelevant. Whether arterial blockage can be prevented says nothing as to what whether it was moderate exercise that prevented the blockage of the arteries.


Hi, I got this right, but if this question was asking for a necessary condition, would (B) qualify as one or would it merely be a premise booster (inferred from the 1st sentence, "since anything that lowers blood cholesterol levels...lowers the risk of arterial blockage due to blood clots)

Thx.
User avatar
 
ManhattanPrepLSAT1
Thanks Received: 1909
Atticus Finch
Atticus Finch
 
Posts: 2851
Joined: October 07th, 2009
 
This post thanked 2 times.
 
 

Re: Q18 - Moderate exercise lowers the risk

by ManhattanPrepLSAT1 Fri Apr 05, 2013 8:53 pm

tz_strawberry Wrote:What does this ";" represent??
Maybe a small issue but wonder if it has some meaning...

Thank you

It typically means that what follows serves to support/clarify what came before. And this case it's basically the same. The part that follows does support the very first part of the sentence that "moderate exercise lowers the risk of blockage of the arteries due to blood clots."

wj097 Wrote:Hi, I got this right, but if this question was asking for a necessary condition, would (B) qualify as one or would it merely be a premise booster (inferred from the 1st sentence, "since anything that lowers blood cholesterol levels...lowers the risk of arterial blockage due to blood clots)

Good point, the conclusion does rely on the fact that the risk of blockage of the arteries due to blood clots can be reduced. It does represent an necessary assumption of the argument.

Good points, both of you!
 
ban2110
Thanks Received: 0
Jackie Chiles
Jackie Chiles
 
Posts: 31
Joined: August 18th, 2012
 
 
trophy
Most Thankful
 

Re: Q18 - Moderate exercise lowers the risk

by ban2110 Fri Jul 05, 2013 1:56 pm

Hallo,

I'm still a little unsure about why (D) is the correct answer. The stimulus states: "if the data reported in a recent study are correct" and so I took answer choice D as a mere restatement of this fact. I eliminated it because it seemed to me like a premise booster. Can someone please clarify why it is not a premise booster.

Also, I diagrammed this problem a little differently than the explanations given above and would very much appreciate it if someone could tell me if I'm at least on the right track in terms of diagramming or if I'm completely off the mark?

Diagram Abbre. LBC = Lowers Blood Cholesterol, LRB = Lowers Risk of Blockage, DC = Data Correct, ME = Moderate Exercise

Premises
LBC --> LRB
DC --> LBC
(I then "consolidated these two diagrams as: DC --> LRB)

conclusion
ME --> LRB

So I then concluded that I needed to find an assumption that ties DC to ME. However, I was unable to find that. I ended up choosing (D) only because it was the only one that mentioned data and I was sort of panicked at that point.

Thank you kindly!
 
monygg85
Thanks Received: 1
Jackie Chiles
Jackie Chiles
 
Posts: 29
Joined: December 04th, 2012
 
 
 

Re: Q18 - Moderate exercise lowers the risk

by monygg85 Thu Sep 12, 2013 5:11 pm

Well the way I saw it is that,

if DC (data correct) then MELBC (moderate exercise lowers blood cholesterol levels). DC ---> MELBC

If MELBC then LHA (lowers risk of hardening of arteries).
MELBC --> LHA

If LHA then LABBC (lower risk of arterial blockage due to blood clots). LHA --> LABBC

CONCLUSION: MELBC ---> LABBC

Well from above we can see this conclusion can follow, logically, if DC is assumed. B/c DC leads to MELBC, which from following down the chain of logic leads to LABBC.

In other words assuming DC leads us all the way down to a proper conclusion of LABBC

Hope that helped some!
User avatar
 
WaltGrace1983
Thanks Received: 207
Atticus Finch
Atticus Finch
 
Posts: 837
Joined: March 30th, 2013
 
 
trophy
Most Thanked
trophy
Most Thankful
trophy
First Responder
 

Re: Q18 - Moderate exercise lowers the risk

by WaltGrace1983 Mon May 05, 2014 7:07 pm

ban2110 Wrote:Hallo,I'm still a little unsure about why (D) is the correct answer. The stimulus states: "if the data reported in a recent study are correct" and so I took answer choice D as a mere restatement of this fact. I eliminated it because it seemed to me like a premise booster. Can someone please clarify why it is not a premise booster.


It is not a premise booster because the nature of both statements are completely different.

I think what may be confusing people is that the question is never giving us anything definite! It is merely a long list of conditionals - "if THIS happened then THIS would happen." The argument isn't saying "This happened, therefore THIS happened, therefore THIS happened." Let me further explain by breaking down the entire argument.

"Moderate exercise lowers the risk of blockage of the arteries."

    This is our conclusion.
    ME → (↓ Blockage of Arteries)


"...since anything that lowers blood cholesterol levels also lowers the risk of hardening of the arteries...lowers the risk of arterial blockage due to blood clots."

    This is just a long premise.
    (↓ Blood Cholesterol) → (↓ Hardening) → (↓ Blockage of Arteries)


"...and, if data reported...correct, moderate exercise lowers blood cholesterol levels."
    This is another premise.
    Correct → (ME → (↓ Blood Cholesterol))


So now we have two premises and one conclusion. The important thing to note is, as I said, we have absolutely NO definitive pieces of information. All we have is a long strand of conditionals. We don't know anything if the triggers of these conditionals don't happen.

The next step would be to link up what we do have.

The first thing I see is that we can very neatly and clearly link up the two premises (great!)

    Correct → (ME → (↓ Blood Cholesterol)) → (↓ Hardening) → (↓ Blockage of Arteries)


*Now keep in mind, we don't know anything about the study and whether or not it is correct!!!

Now let's look back at the conclusion!

    ME → (↓ Blockage of Arteries)


Take a hard look at the conclusion and the premises? What do you notice?

    Conclusion: ME → (↓ Blockage of Arteries)


    Correct → (ME → (↓ Blood Cholesterol)) → (↓ Hardening) → (↓ Blockage of Arteries)


Gah! The conclusion is already perfectly stated in the premises! The linked up conditionals do actually match our conclusion! So what do we do? Well we have to realize that this argument is something we have to have properly drawn. So our task is actually quite easy! Just show that the study is correct!

Why? Because we know that IF the study is correct, THEN the rest of the stuff will follow (i.e. that ME actually does lead to a lowered risk of arterial blockage!)

This is expressed in (D).

    (A) Yes but what does it SAY about the relationship? This seems to be necessary but not sufficient? However, I am still doubtful that its even necessary because the premises outwardly says that ME leads to lowered blood cholesterol levels. It might not have had to investigate the relationship per se.
    (B) But what about the study? How do we prevent the blockage?
    (C) Premise booster.
    (D) Correct.
    (E) We know what happens if the risk of hardening is DECREASED but we don't know anything about what happens if the arteries actually harden.


if your doing this in real time, just link up the premises, write out the conclusion, see where the gap is. Done!
 
Jdanz653
Thanks Received: 0
Vinny Gambini
Vinny Gambini
 
Posts: 13
Joined: August 05th, 2015
 
 
 

Re: Q18 - Moderate exercise lowers the risk

by Jdanz653 Sun Aug 09, 2015 5:52 pm

If the conclusion stated that moderate exercise could possibly lower the risk of blockage (a less definitive statement), would it have been valid?
 
abdelmalak17
Thanks Received: 0
Vinny Gambini
Vinny Gambini
 
Posts: 6
Joined: December 22nd, 2015
 
 
 

Re: Q18 - Moderate exercise lowers the risk

by abdelmalak17 Wed Mar 02, 2016 5:03 pm

I got the question correct, but spent too much time trying to prove to myself why answer choice D was correct. I could have spent way less time if I didn't get confused on breaking down the argument. I chose D, because the term: "If the data reported in the study is correct," was something new and most of the time on difficult sufficient questions the new term will likely be the correct answer choice, also chose it because it was a pretty strong statement.

Can someone break down the argument and show me the premises and the conclusion? I know "since" can have a premise, conclusion. Perhaps that's why I confused myself thinking that "since anything that lowers blood cholesterol levels also lowers the risk of hardening of the arteries (premise), "which" in turn lower the risk of arterial blockage due to blood clots (Conclusion?); and,....

THANKS IN ADVANCE!
User avatar
 
tommywallach
Thanks Received: 468
Atticus Finch
Atticus Finch
 
Posts: 1041
Joined: August 11th, 2009
 
This post thanked 1 time.
 
 

Re: Q18 - Moderate exercise lowers the risk

by tommywallach Mon Mar 07, 2016 10:06 pm

Matt actually did that up top, using formal notation. The stuff above the dotted line are all premises. The thing below the line is the conclusion. Does that help?

-t
Tommy Wallach
Manhattan LSAT Instructor
twallach@manhattanprep.com
Image
 
OliviaM54
Thanks Received: 0
Vinny Gambini
Vinny Gambini
 
Posts: 3
Joined: October 08th, 2019
 
 
 

Re: Q18 - Moderate exercise lowers the risk

by OliviaM54 Wed Oct 23, 2019 1:42 pm

I wanted to try explaining answer choice D). Please let me know if this makes any sense.

Conclusion is in the first sentence i.e. that moderate exercise lowers risk of artery blockage due to blood clots
Premises:
1) Lowers blood cholesterol levels --> lowers risk of hardening arteries
2) Lowers risk of hardening arteries --> lowers risk of arterial blockage due to blood clots
3) Studies CORRECT --> moderate exercise lowers blood cholesterol levels
But the author already made the conclusion that moderate exercise lowers risk of artery blockage and so the study MUST BE correct. Why? If the studies are not correct, the argument falls apart because when we fail the sufficient, the premise goes away. So, that means there is no premise connecting moderate exercise to lowering blood cholesterol levels, and thus making the author's conclusion (in the first sentence) unsubstantiated.